1. In the four graphs below, determine what transformation was used to transform AABC
to AA'B'C'. Then, for each one, write the
correct description of the transformation
-along with the algebraic notation

the graph is in the image

1. In The Four Graphs Below, Determine What Transformation Was Used To Transform AABCto AA'B'C'. Then,

Answers

Answer 1

Answer:

first would be a translation of (2,-3) second one is another 90 degree rotation counter clockwise.


Related Questions

I am thinking of a number. The sum of its digits is divisible by 2. My number is a multiple of 11. My number is greater than 4 x 5 but less than 7 x 8+ 23. The number is a multiple of 3. What is my number? Is it possible to have more than one answer? ​

Answers

Answer:

66

Step-by-step explanation:

Number:xy

20<xy<79

Multiples of 2,3,11

The least common multiple of this number is 66.

Tiana bought 3 video games that each cost
the same amount. The tax on each video
game was $1.29. She paid a total of $68.37
after tax. What was the price of each video
game, before tax?
a

Answers

well, we know she paid a total after tax of $68.37, we also know that each game had a tax of $1.29, so 3 games that'll be 3 * 1.29 = 3.87.

so if we take out the tax from 68.37, that'll be 68.37 - 3.87 = 65 bucks.

we know each game cost the same amount so 65 ÷ 3 ≈ 21.67 before tax.

Tiana bought 3 video games at a cost of $22.36 each, before tax. The total amount paid after tax was $68.37.

Given that,

Tiana bought 3 video games.

The cost of each video game was the same.

The tax on each video game was $1.29.

Tiana paid a total of $68.37 after tax.

To find the price of each video game before tax,

Start by subtracting the total tax from the total amount paid.

Tiana paid a total of $68.37 after tax and the tax on each video game was $1.29.

So, subtract the tax from the total amount to get the price before tax.

$68.37 - $1.29 = $67.08.

Since Tiana bought 3 video games,

Divide the total price before tax by 3 to find the price of each video game.

$67.08 ÷ 3 = $22.36.

Hence, the price of each video game before tax was $22.36.

To learn more about subtraction visit:

https://brainly.com/question/17301989

#SPJ3

In the figure below, triangle ABC undergoes a reflection and a translation to become triangle PQR. In triangle ABC, m

The angle measures of triangle PQR are: m

Answers

Answer:

Step-by-step explanation:

A corresponds to angle P, so angle P measues 70 degrees.

Similarly, angle Q measures 60 degrees and angle R measures 50 degrees.

Two rectangles share a common side as shown.
Determine whether each expression can be used to determine the
total area, in square feet, of the two combined rectangles.
ft
Select Yes or No for each expression.
7.52 + 20
o Yes
Ο Νο
*
7.5 ft
x ft
27.5 + 2
Yes
Ο Νο
20 (7.5 + x)
o Yes
Ο Νο
150 + 3
Yes
Ο Νο
202 + 150
O Yes
No

Answers

Answer:

yes they are

Step-by-step explanation:

Question 1-5
Sarah is demonstraing the idea that polynomials are closed under addition. Her work and explanation are shown.
Select an equation and phrases to complete the statement.
Part A:
Add 3x4 + 6x - 7 to
Equation A 2x - (2x)2 + 4x-1
Equation B (2x)2 + 4.4x - 9
Equation C (-8x)-2 - (3x)3 +7
Part B:
This is closed because
of the sum are
and the sum is a polynomial.

Answers

This is closed under addition because the sum of the polynomial produces another polynomial.

What is a polynomial?

A polynomial is an expression that involves the addition, subtraction and multiplication of variables.

Polynomials will be closed under an operation if the operation produces another polynomial.

(3x⁴ + 5x - 7) + (2x - (2x)² + 4x - 1) + [(-8x)⁻² - (3x)³ + 7]

= 3x⁴ - 27x³ - 4x² + 9x - 1 + (-8x)⁻²

This is closed under addition because the sum of the polynomial produces another polynomial.

Find out more on polynomial at: https://brainly.com/question/2833285

The first term of a geometric sequence is 4 and grows exponentially by a factor of 3. Murphy writes out the terms and says that the sum of the 4th and 5th terms is​ 1,296. Explain​ Murphy's error and correct it. (Fill in the blanks)
Murphy added terms
____
together. The correct sum of the 4th and 5th terms is ___
​(Simplify your​ answer.)

Answers

The geometric sequence can increase or decrease exponentially

The correct sum of the 4th and the 5th terms is 432

How to determine the correct sum?

The given parameters are:

Initial value (a) = 4

Growth factor (r) = 3

The nth term of a geometric sequence is calculated as:

[tex]T_n = ar^{n-1}[/tex]

This gives

[tex]T_n = 4 * 3^{n-1}[/tex]

For the 4th term; we have:

[tex]T_4 = 4 * 3^{4-1}[/tex]

[tex]T_4 = 108[/tex]

For the 5th term, we have:

[tex]T_5 = 4 * 3^{5-1}[/tex]

[tex]T_5 = 324[/tex]

Add the 4th and the 5th terms

Sum = 108 + 324

Sum = 432

Hence, the correct sum of the 4th and the 5th terms is 432

Read more about geometric sequence at:

https://brainly.com/question/24643676

Four eighths of kelly’s beads are red, 18 are white, and the rest are blue.
what fraction of kelly’s beads are blue?

Answers

4/8=1/2 so Kelly blue beads are also 4/8. Sorry if I’m wrong.


Fifteen less than the square of a number is the same
as twice the number. Find the number.

Answers

Call the number : a

We have : [tex]a^{2} - 15 = 2a[/tex]
[tex]- > a^{2} - 2a = 15 - > a(a-2) = 15[/tex]

Now, we list all the numbers that can be multiplied to get 15 :

1 x 15 (and reverse)
3 x 5 (and reverse)

-1 x (-15) (reverse)
-3 x (-5) (a l s o r e v e r s e)

We substitute in :

If a = 1 -> a(a-2) = 1 x (-1) = -1 (not equal 15)
If a = 3 -> a(a-2) = 3 x 1 = 3 (not equal 15)
If a = -1 -> a(a-2) = -1 x (-3) = -3 (also not equal 15)
And if a = -3 -> a(a-2) = -3 x (-5) = 15 (satisfied)

So, the number is -3

Recheck : -3(squared) - 15 = 9 - 15 = -6
Twice of -3 = -6

Hello!

[tex]========================================[/tex]

First of all, let the unknown number be z.

Then, "the square of z" means "z squared" which is the same as "z times z" or just z².

Now, "fifteen less than the square of z" means we subtract 15 from z²:

z²-15

Then, this equals twice the number, or 2 times z:

z²-15=2z

[tex]========================================[/tex]

Notes:-Hope everything is clear.Let me know if you have any questions!Always remember: [tex]\boxed{Knowledge~is~power!}[/tex]Enjoy your day!Answered by[tex]\boxed{An~Emotional~Helper}[/tex]

:-)

The height of the prism is the Blank between the bases.

Answers

Answer:

so how do i find the earea

Step-by-step explanation:

can you pls help me i need it?

Answers

Answer:

$114.75

Step-by-step explanation:

You have to multiply the hours by the wage. Looking at the time that the worker was in during the morning, it was a total of 4 hours. Since the wage is $13.50/hr, we would multiply 13.50 by 4.

13.50 * 4 = 54.00

So, now we have to add together the total hours in the afternoon. If we count the time, we get 4 1/2 hours. So, now we multiply 13.50 by 4.5.

13.50 * 4.5 = 60.75

Now, to find the total pay for that day, we add both the morning and the afternoon pay together.

54.00 + 60.75 = 114.75

Therefore, the pay for this day is $114.75.

Pls help me
AND

PLS FOLLOW THE RULES

TROLL FOR POINTS= REPORT

FIRST AND CORRECT ANSWER= BRAINLIEST

( NO LINKS ) at all

Answers

Pythagorean Theorem: a^2 + b^2 = c^2

a = 23 m

b = 11 m

c^2 = (23 m)^2 + (11 m)^2 = 650 m^2

c = sqrt(650 m^2) = 25.5 (rounded to nearest tenth)

Jill bought 3 pints of blueberries for $3.02 each. What is the best estimate of how much Jill spent

Answers

Answer: 9.06

Step-by-step explanation:

Do; 3.02+3.02+3.02

This adds up to 9.06

Find the value of log 1 2 462 to four decimal places. –8.8517 –0.1130 0.1130 8.8517

Answers

The logarithm value of log 1/2 462 to four decimal places is  –8.8517.

What is a logarithm?

The logarithm is the mathematical inverse function to exponentiation.

The logarithm of a number x is the exponent to another fixed term.

It is given the log of 462 to a base of 1/2.

[tex]log_{0.5} 462 = \frac{log462}{log0.5}[/tex]

 = –8.8517

Therefore, the logarithm value of log 1/2 462 to four decimal places is  

–8.8517.

Learn more about logarithm :

https://brainly.com/question/3181916

PLEASE HELP ME! THERE IS A PICTURE

Answers

B. 1/40…. You divide 1/2 by 20 to find out how much hand sanitizer was used on each desk. You can keep change flip meaning keep the 1/2 change the division to multiplication and flip the 20 to 1/20 then multiply. 1/2 by 1/20 which equals 1/40.

Answer:

I think 10

letter C.

Step-by-step explanation:

divide *2

Use the dot plot to answer the question.

A dot plot that has 3 dots at 1, 4 dots at 2, 3 dots at 3, 4 dots at 4, 5 dots at 5, & 3 dots at 6.

What is the mean of the data set in the dot plot?

Enter your answer as a number rounded to the nearest tenth, like this: 42.5

Answers

The mean of the data set is 2.20

Prism M is a dilation of Prism N. The height of Prism M is 4 3/4 ft, and the volume of Prism M is 68 2/5 ft³. The height of Prism N is 2 3/8 ft. What is the volume of Prism N?Enter your answer as a mixed number in simplest form by filling in the boxes.

Answers

Answer:

  8 11/20 ft³

Step-by-step explanation:

For dilated figures, the ratio of volumes is the cube of the ratio of corresponding linear dimensions.

  Vn/Vm = (Hn/Hm)³

  Vn = Vm(Hn/Hm)³ = (68 2/5 ft³)((2 3/8 ft)/(4 3/4 ft))³

  Vn = (342/5 ft³)((19/8)/(19/4))³ = (342/5 ft³)(1/2)³ = 342/40 ft³

  Vn = 8 11/20 ft³

The volume of Prism N is 8 11/20 ft³.

Answer:

The answer is 8 11/20.

Pls help dont need an explanation just an answer

Answers

At y-intercept x=0.
Y-intercept = 1

(0,1). & (-1 ,-4)

Slope =y2-y1/(x2-c1)
=(-4-1)/(-1-0)
=-5/-1
Slope =5
= (

Answer:

[tex]\boxed{\text{Slope = 5}}[/tex]

[tex]\boxed{\text{y-intercept = 1}}[/tex]

Step-by-step explanation:

To find the slope of the line, we must pick any two points on the line. Using those two points, we will calculate the Rise and the Run. Then, we will use the slope formula (Rise/Run) to find the slope. The y-intercept is the intersection of the point on the y-axis.

[tex]\text{My chosen points: (0,1) and (1,6)}[/tex]

[tex]\text{Rise = 5; Run = 1}[/tex]

≡≡≡≡≡≡≡≡≡≡≡≡≡≡≡≡≡≡≡≡≡≡≡≡≡≡≡≡≡≡≡≡≡≡≡≡≡≡≡≡≡≡≡≡≡≡≡≡≡≡≡≡≡

[tex]\text{Slope}= \dfrac{\text{Rise}}{\text{Run}}[/tex]

➡ [tex]\bold{Slope = \frac{5}{1} = \underline{5}} \ \ \ \ \ \ \ \ \ \ \ \ \ \ \ \ \ \ \ \ \ \ \ \ \ \ \ \ \ \ \ \ \ \ \ \ \ \ \ \ \ \ \ \ \ \ \ \ \ \ [\text{Rise = 5; Run = 1}][/tex]

[tex]\underline{........................................................................................................................}[/tex]

[tex]\text{Intersection on y-axis:} \ (0,1)[/tex]

➡ [tex]\text{y-intercept} \rightarrow (0,\bold{ 1}) \rightarrow \bold{\underline{{1}}}}[/tex]

≡≡≡≡≡≡≡≡≡≡≡≡≡≡≡≡≡≡≡≡≡≡≡≡≡≡≡≡≡≡≡≡≡≡≡≡≡≡≡≡≡≡≡≡≡≡≡≡≡≡≡≡≡

Help is number 2 correct and help for number one​

Answers

Number 2 is incorrect :

[tex]\frac{-25x^{3}+15x }{5x} = \frac{-25x^{3}}{5x} + \frac{15x}{5x} = -5x^{2} + 3[/tex]

Number 1 :

[tex](-5)^{3} . 13^{-2} . (-5) .13^{9} = -125 . \frac{1}{13^{2}} .(-5).13^{9} = 625 . \frac{13^{9}}{13^{2}} = 625 . 13^{7}[/tex] (i dont think we can bare calculate this cause it;s too big ;-;)

Volt Electronics sells a Y-Box game console for $207. The store markup on thr Y-Box is 38%. What was the wholesale price that the store paid for it?​

Answers

The wholesale price that the store paid for the Y-Box game is $150.

What is the wholesale price?

When the price of an item is marked up, it means that the price of the item was increased by a certain percentage. The good then becomes more expensive.

Wholesale price = selling price / ( 1 + percentage mark up)

$207 / (1+ 38)

$207 / (1.38) = $150

To learn more about profit, please check: https://brainly.com/question/26181966

hello, me again polease help me with this, and show work

Answers

Answer:

Hope it helps u

Step-by-step explanation:

1.7

1.749

1.75

1.775

1.796

1.84

Answer:

the order would go 1.7, 1.749, 1.75, 1.775, 1.796, then 1.84.

Step-by-step explanation:

becuase if you ignore how long the numbers are, and focus on numerical order like 1, 2, 3, and apply that to this, this becomes the order.


Given the equation of a circle. X^2+ y^2 - 6x + 2y +1 = 0, find the coordinates of the center and the length of the radius.
Coordinates for the center=
Length of the radius =

Answers

Answer:

I know the answer but you may select me as Brainliest

Answer:
Center: (3, -1)
Radius: 3

Steps:
Rewrite in standard form to find the center
(h,k) and radius.

x^2 + y^2 - 6x + 2y + 1 = 0
Standard Form: (x - 3)^2 + (y + 1)^2 = 9

Center: (3, -1)
Radius: 3

In the figure below, mR is 66°, and mT is 130°. Note: Figure is not drawn to scale. What is mQ? A. 24° B. 50° C. 64° D. 116°

Answers

Answer:

b

Step-by-step explanation:

i just think so correct me if im wrong

What points should I circle and in which point do they intersect?

I WILL MARK BRAINLIEST AND GIVE THANKS

Answers

Answer:

[tex]\displaystyle [3, -2][/tex]

Explanation:

All you do is substitute three in for [tex]\displaystyle x[/tex]to get the y-coordinate of [tex]\displaystyle -2.[/tex]This means that these equations meet at the ordered pair of [tex]\displaystyle [3, -2].[/tex]

I am joyous to assist you at any time.

A triangular pyramid has a base shaped like an equilateral triangle. The legs of the equilateral triangle are all 1 yard long, and the height of the equilateral triangle is 0.9 yards. The pyramid's slant height is 1 yard. What is its surface area?

Answers

Answer:

base: 1/4 * 12^2 * √3 = 36√3

each face: 1/2 * 12 * 8 = 48

Now finish it off

Step-by-step explanations:

What are the y-intercept and the horizontal asymptote of g(x) = 4x + 3?
(0, 3); y = 4
(0, 4) ; y = 3
(0, 5) ; y = 4
(0, 7) ; y = 3

Answers

Using exponential function concepts, it is found that that the y-intercept and the horizontal asymptote are given by, respectively:

(0, 3); y = 4

What is an exponential function?

It is modeled by:

[tex]y = ab^x + c[/tex].

In which:

a is the initial value.b is the rate of change.y = c is the horizontal asymptote.

In this problem, the equation is:

[tex]y = 4^x + 3[/tex].

Hence the horizontal asymptote is y = 3, and the y-intercept is y when x = 0, hence:

[tex](0,y) = (0, 4^0 + 3) = (0, 4)[/tex]

Which means that the first option is correct.

More can be learned about exponential function concepts at https://brainly.com/question/25537936

Answer:

B. (0, 4) ; y = 3

Step-by-step explanation:

I took the exam for algebra 2

graph "-3n+4<25" on a line plot

Answers

Answer:The solution is in the attached file

Step-by-step explanation:

name 2 semi circles


Answers

Answer:

Arc CDE and arc EAC.

Step-by-step explanation:

Semicircles are exactly one-half of a circle and are always equal to 180 degrees. Since line CE is a horizontal line, measuring 180 degrees, 2 semicircles can be arcs CDE and EAC because they equal half of the circle.

I hope this helps!

helpppp :) 50 points
An equation is shown below:

4(2x – 5) = 4

Part A: How many solutions does this equation have? (4 points)

Part B: What are the solutions to this equation? Show your work. (6 points)

Answers

Answer:

x = 3

Step-by-step explanation:

A only 1 solution

B

4(2x - 5) = 4 ( divide both sides by 4 )

2x - 5 = 1 ( add 5 to both sides )

2x = 6 ( divide both sides by 2 )

x = 3

Answer:

[tex]\boxed{\sf{x=3}}[/tex]

Step-by-step explanation:

Isolate it by the term of x from one side of the equation.

Part A. There will be one solution.Part B.

4(2x-5)=4

First, divide by 4 from both sides.

[tex]\sf{\dfrac{4\left(2x-5\right)}{4}=\dfrac{4}{4}}[/tex]

Then, solve.

4/4=1

Rewrite the problem down.

2x-5=1

Add by 5 from both sides.

[tex]\sf{2x-5+5=1+5}[/tex]

Solve.

1+5=6

2x=6

Divide by 2 from both sides.

2x/2=6/2

Solve.

Divide the numbers from left to right.

6/2=3

x=3

Therefore, the correct answer is x=3.

I hope this helps you! Let me know if my answer is wrong or not.

Use your TI-83 to determine the necessary t* value for a 90% confidence interval
based on a sample size of 55 and a sample standard deviation of 7.42.

2.005
1.669
1.96
1.645
2.67

Answers

Answer:

Step-by-step explanation:

It is 1.645. My calculator is a little off though; see attached picture.

The rule for the number of fish in a home aquarium is 1 gallon of water for each inch of fish length. Marta's aquarium holds 39 gallons of water and Hank's aquarium hold 51 gallons of water. The aquarium holds two types of fish, fish A and fish B. If Marta bought 3 of fish A and 2 of fish B, and Hank bought 3 of fish A and 4 of fish B, how long is fish A and how long is fish B?

Answers

Answer: Fish A: 9 inches ; Fish B: 6 inches

Step-by-step explanation:

This can be turned into a systems of equation, where x equals the length of fish A and y equals the length of fish B:

3x + 2y = 39 → -3x - 2y = -39

3x + 4y = 51 → 3x +4y = 51

Combine: 0 + 2y = 12 → y = 6 inches.

Plug back in: 3x + 2(6) = 39 → 3x = 27 → x = 9 inches.

Other Questions
need help pleaseeeeeee An object absorbs red, orange, yellow, blue, indigo, and violet light. What color is it?A. blueB. greenC. indigoD. red Which excerpt from Fast Food Nation best illustrates the use of the rhetorical appeal pathos?O Teenagers have long provided the fast food industry with the bulk of its workforce. The strict regimentation at fast food restaurants creates standardized products.O Although Richard and Mac McDonald introduced the division of labor to the restaurant business, it was a McDonald'sexecutive named Fred Turner who created a production system of unusual thoroughness and attention to detail. But the stance of the fast food industry on issues involving employee training, the minimumwage, labor unions, andovertime pay strongly suggests that its motives in hiring the young, the poor, and the handicapped are hardlyaltruistic. What are the top 4 expenses younger millennials pay for using P2P? Balance the chemical equation At a window poem what does line 7 mean asey wants to buy some instruments for his niece and nephew. He has $40 to spend. Based on the prices listed below, what is the estimated total, rounded to the nearest dollar? Which explains why he can afford the tambourine and xylophone?Tambourine: $13.95Xylophone $23.50The estimated total to the nearest dollar is $38, so he can afford them.The estimated total to the nearest dollar is $37, so he can afford them.The estimated total to the nearest dollar is $36, so he can afford them. The estimated total to the nearest dollar is $37.45, so he can afford them. What connection does the author draw between branding and human behavior? Battle of Brands A large box of strawberris vaires in price.the cost of a box is never graeater than or equal to 10$ the lowest price ever reached for the box of strawberries is 5$. write an inquality to show all the possible prices (P) for a box of strawberries What are the qualifications to be an eligible voter in Georgia? Which one is NOT TRUE about gravity?Gravity attracts all objects to other objects.Gravity attracts all objects to other objects.Objects do have to touch each other for gravity to affect them.Objects do have to touch each other for gravity to affect them.Gravity is a pulling force that works across spaceGravity is a pulling force that works across spaceGravity makes objects fall. Gravity makes objects fall. What are the solutions to the equation 30/x=6 The rights of citizens were violated by the u. S. Government during world war ii when it established. Help me please Im struggling with this subject :/ PointM is the midpoint of AB. if the coordinates of A are (6,-4) and the coordinates of M are (4,2), what are the coordinates of B? Diana can earn money for the tickets she sells. Which of the following statements describes the variables in this situation correctly? The amount of money earned is the independent variable because it affects the number of tickets sold. The amount of money earned is the dependent variable because it affects the number of tickets sold. The number of tickets sold is the independent variable because it affects the amount of money earned. The number of tickets sold is the dependent variable because it affects the amount of money earned. Jean listed the steps involved in the life cycle of a gymnosperm. 1 - The pollen gets stuck in a sticky liquid. 2 - Sperm from the pollen fertilize the egg in the ovule. 3 - The ovule develops into a seed. 4 - When mature, the seeds are dispersed. Which step is missing? Before step 3, the ovule develops cones. Just after step 3, the ovule develops cones. Before step 1, wind carries pollen from the male to the female cones. Just after step 1, wind carries pollen from the male to the female cones A drum is struck, producing a wave with a wavelength of 110 cm and a speed of 2.42 104 m/s. What is the frequency of the wave? What is the period? Graph the line with the equation y=1/2x+5 What does the phrase "made new lanterns out of their hearts and minds (paragraph 3)represent?O A. the children's artistic abilityB. the children's vision of beautyO C. the children's understanding of designO D. the children's cultural beliefs